Administración     

Olimpiadas de Matemáticas
Página de preparación y problemas

Selector
La base de datos contiene 1154 problemas y 775 soluciones.
OME Local
OME Nacional
OIM
OME Andalucía
Retos UJA
Problema 788
Hallar todas las soluciones reales de la ecuación \[3^{x^2-x-y}+3^{y^2-y-z}+3^{z^2-z-x}=1.\]
pistasolución 1info
Pista. Usa la desigualdad entre las medias aritmética y geométrica para probar que se cumple la desigualdad $\geq$ y analiza en qué casos se tiene una igualdad.
Solución. La desigualdad entre las medias aritmética y geométrica nos dice que \begin{align*} \frac{3^{x^2-x-y}+3^{y^2-y-z}+3^{z^2-z-x}}{3}&\geq\sqrt[3]{3^{x^2-x-y}\cdot 3^{y^2-y-z}\cdot 3^{z^2-z-x}}\\ &=3^{\frac{x^2-2x+y^2-2y+z^2-2z}{3}}=3^{\frac{(x-1)^2+(y-1)^2+(z-1)^2-3}{3}}\\ &=\tfrac{1}{3}\cdot 3^{\frac{(x-1)^2+(y-1)^2+(z-1)^2}{3}}\geq \frac{1}{3}. \end{align*} Por lo tanto, $3^{x^2-x-y}+3^{y^2-y-z}+3^{z^2-z-x}\geq 1$ para todo $x,y,z\in\mathbb{R}$ y, si la igualdad se alcanza, tiene que ser $(x-1)^2+(y-1)^2+(z-1)^2=0$, es decir, $x=y=z=1$. Como $x=y=z=1$ verifica la ecuación del enunciado, deducimos que esta es la única solución.

Nota. Las exponenciales pueden ocultar la aplicación de la desigualdad entre las medias aritmética-geométrica, pero una solución similar se tiene aplicando la desigualdade de Jensen a la función convexa $f(t)=3^t$. ¿Sabrías escribir los detalles?

Si crees que el enunciado contiene un error o imprecisión o bien crees que la información sobre la procedencia del problema es incorrecta, puedes notificarlo usando los siguientes botones:
Informar de error en enunciado Informar de procedencia del problema
Problema 785
Encontrar todas las soluciones reales del sistema de ecuaciones \[\left\{\begin{array}{l} y^3-6x^2+12x-8=0,\\ z^3-6y^2+12y-8=0,\\ x^3-6z^2+12z-8=0.\end{array}\right.\]
pistasolución 1info
Pista. Cada una de las ecuaciones se parece mucho a $(t-2)^3=t^3-6t^2+12t-8$.
Solución. Consideremos la función \[f:\mathbb{R}\to\mathbb{R},\qquad f(t)=\sqrt[3]{6t^2-12t+8},\] con la que el problema se reduce encontrar $x,y,z\in\mathbb{R}$ tales que $y=f(x)$, $z=f(y)$ y $x=f(z)$, es decir, empezando en $x$ queremos volver a obtener $x$ tras aplicar tres veces la función. Observemos que tiene que ser $x\geq\sqrt[3]{2}$ ya que se debe cumplir que $x^3=6z^2-12z+8=6(z-1)^2+2\geq 2$. También tenemos que $f(x)=x$ se traduce en que $(x-2)^3=x^3-6x^2+12x-8=0$, luego $x=2$ es el único valor que cumple $f(x)=x$. Distingamos casos:
  • Si $\sqrt[3]{2}\leq x\lt 2$, entonces $(x-2)^3\leq 0$, luego $x^3\lt 6x^2+12x-8$ y, tomando raíces cúbicas, $x\lt f(x)$. Además, se tiene que $f(x)^3=6(x-1)^2+2\leq 6(2-1)+2=8$ (ya que esta parábola tiene su máximo en el $x=2$, el punto del intervalo $[\sqrt[3]{2},2]$ más alejado del vértice $x=1$). Deducimos que en este caso se cumple que \[\sqrt[3]{2}\leq x\lt f(x)\lt 2,\] luego no existen $y,z\in\mathbb{R}$ tales que $y=f(x)$, $z=f(y)$ y $x=f(z)$ puesto que tendríamos que $x\lt f(x)= y\lt f(y)= z\lt f(z)= x$ y esto es contradictorio.
  • Si $x\gt 2$, el razonamiento es parecido pero un poco más sencillo. Como en este caso $(x-2)^3\gt 2$, obtenemos directamente que $x^3\lt 6x^2-12x+8$ y, por tanto, $f(x)\gt x\gt 2$, lo que impide también la existencia de $y,z\in\mathbb{R}$ tales que $y=f(x)$, $z=f(y)$ y $x=f(z)$.

La única posibilidad que nos queda es $x=2$, que nos da $y=f(x)=f(2)=2$ y $z=f(y)=f(2)=2$. En consecuencia, $x=y=z=2$ es la única solución del sistema.

Si crees que el enunciado contiene un error o imprecisión o bien crees que la información sobre la procedencia del problema es incorrecta, puedes notificarlo usando los siguientes botones:
Informar de error en enunciado Informar de procedencia del problema
Problema 781
Encontrar todas la soluciones reales $(x,y)$ del sistema de ecuaciones \[\left\{\begin{array}{l}x^2-xy+y^2=7,\\x^2y+xy^2=-2.\end{array}\right.\]
pistasolución 1info
Pista. Transforma el sistema en otro sistema con incógnitas $s=x+y$ y $p=xy$.
Solución. Si consideramos las nuevas variables $s=x+y$ y $p=xy$ (suma y producto), el sistema se puede reescribir como \[\left\{\begin{array}{l}s^2-3p=7,\\sp=-2.\end{array}\right.\] Despejamos $p=\frac{s^2-7}{3}$ en la primera ecuación y sustituimos en la segunda para obtener $(s^2-7)s=-6$ o, lo que es lo mismo, $s^3-7s+6=0$. Por Ruffini obtenemos rápidamente la factorización $s^3-7s+6=(s-1)(s-2)(s+3)$, lo que nos da tres posibilidades:
  • Si $s=1$, entonces $p=\frac{s^2-7}{3}=-2$. Ahora bien, conociendo la suma y el producto, las incógnitas originales $x$ e $y$ son las soluciones de la ecuación $z^2-sz+p=0$. En este caso, esta última ecuación es $z^2-z-2=0$, que tiene soluciones $z=-1$ y $z=2$, lo que nos da las soluciones $(x,y)=(-1,2)$ y $(x,y)=(2,-1)$.
  • Si $s=2$, entonces $p=\frac{s^2-7}{3}=-1$, luego $x$ e $y$ son las soluciones de $z^2-2z-1=0$, que son $z=1\pm\sqrt{2}$. Esto nos da otras dos soluciones al sistema: $(x,y)=(1+\sqrt{2},1-\sqrt{2})$ y $(x,y)=(1-\sqrt{2},1+\sqrt{2})$.
  • Si $s=-3$, entonces $p=\frac{s^2-7}{3}=\frac{2}{3}$, luego $x$ e $y$ son las soluciones de $z^2+3z+\frac{2}{3}=0$, es decir, $z=\frac{-1}{6}(9\pm\sqrt{57})$. Esto nos da las dos últimas soluciones al sistema: $(x,y)=(frac{1}{6}(9+\sqrt{57}),frac{-1}{6}(9-\sqrt{57}))$ y $(x,y)=(frac{-1}{6}(9-\sqrt{57}),frac{-1}{6}(9+\sqrt{57}))$.

Esto nos da un total de seis soluciones al sistema.

Nota. En los sistemas de ecuaciones con dos incógnitas que son polinómicos y simétricos (cambiar $x$ por $y$ no afecta al sistema), cambiar a la suma-producto suele simplificar la discusión. En cualquier caso, es muy importante saber que tener la suma y el producto equivale a tener las dos incógnitas a través de la ecuación de segundo grado.

Si crees que el enunciado contiene un error o imprecisión o bien crees que la información sobre la procedencia del problema es incorrecta, puedes notificarlo usando los siguientes botones:
Informar de error en enunciado Informar de procedencia del problema
Problema 777
Los números reales no nulos $a$ y $b$ verifican la igualdad \[\frac{a^2b^2}{a^4-2b^4}=1.\] Encontrar, razonadamente, todos los valores tomados por la expresión \[\frac{a^2-b^2}{a^2+b^2}.\]
pistasolución 1info
Pista. Transforma la igualdad inicial en una ecuación bicuadrática en la incógnita $\frac{a}{b}$.
Solución. Podemos transformar la ecuación dada como sigue: \begin{align*} \frac{a^2b^2}{a^4-2b^4}=1&\ \Longleftrightarrow\ a^2b^2=a^4-2b^4\\ &\ \Longleftrightarrow\ a^4-a^2b^2-2b^4=0\\ &\ \Longleftrightarrow\ \left(\tfrac{a}{b}\right)^4-\left(\tfrac{a}{b}\right)^2-2=0. \end{align*} En el último paso, hemos dividido por $b^4$ de forma que obtenemos una ecuación bicuadrática en la incógnita $\frac{a}{b}$. Usando la fórmula para la ecuación de segundo grado, tenemos que \[\frac{a^2}{b^2}=\frac{1\pm\sqrt{9}}{2}=\frac{1\pm 3}{2}=2.\] Nos hemos quedado con la solución positiva puesto que $\frac{a^2}{b^2}$ no puede ser negativo. Con esto nos basta para hallar el único valor posible de la expresión dada puesto que \[\frac{a^2-b^2}{a^2+b^2}=\frac{\frac{a^2}{b^2}-1}{\frac{a^2}{b^2}+1}=\frac{2-1}{2+1}=\frac{1}{3}.\]

Nota. Este argumento nos dice que todos los números reales que cumplen la condición dada son los que cumplen $a^2=2b^2$, es decir $a=\pm\sqrt{2}b$. Observemos que, para estos números, no se anula el denominador $a^4-2b^4$.

Si crees que el enunciado contiene un error o imprecisión o bien crees que la información sobre la procedencia del problema es incorrecta, puedes notificarlo usando los siguientes botones:
Informar de error en enunciado Informar de procedencia del problema
Problema 776
Se considera la inecuación \[|x-1|\lt ax,\] donde $a$ es un parámetro real.
  1. Discutir la inecuación según los valores de $a$.
  2. Hallar los valores de $a$ para los cuales la inecuación tiene exactamente dos soluciones enteras.
pistasolución 1info
Pista. Resuelve la inecuación teniendo en cuenta las intersecciones de las rectas $y=ax$, $y=x-1$ e $y=1-x$.
Solución. Más abajo se puede ver una representación gráfica de las funciones $|x-1|$ (en rojo) y $ax$ (en azul), para distintos valores de $a$. Como las dos semirrectas que forman la gráfica de $|x-1|$ tienen pendiente $\pm 1$, se ve a priori que las soluciones de la ecuación formarán un intervalo no vacío abierto y acotado para $0\lt a\lt 1$, una semirrecta abierta para $a\gt 1$ o $a\lt -1$ no habrá soluciones para $-1\leq a\leq 0$. Con esto en mente, sólo hay que calcular los puntos de corte de $ax$ con $x-1$ y $1-x$ den función del parámetro $a$, lo que nos da los siguientes resultados (¡compruébalos!):
  • Si $a\lt -1$, las soluciones son los puntos de la semirrecta $(-\infty,\frac{1}{1+a})$.
  • Si $-1\leq a\leq 0$, entonces no hay soluciones.
  • Si $0\lt a\lt 1$, las soluciones son los puntos del intervalo $(\frac{1}{1+a},\frac{1}{1-a})$.
  • Si $a\geq 1$, las soluciones son los puntos de la semirrecta $(\frac{1}{1+a},+\infty)$.
La única forma de que el intervalo de soluciones contenga únicamente dos enteros es que $a\lt 0\lt 1$. En tal caso, tenemos el intervalo $I=(\frac{1}{1+a},\frac{1}{1-a})$, que está contenido en los reales positivos y siempre contiene a $1$, luego queremos saber cuándo $2\in I$ pero $3\not\in I$. Resolviendo las ecuaciones $\frac{1}{1-a}=2$ y $\frac{1}{1-a}=3$, obtenemos que el extremo superior de $I$ es $2$ para $a=\frac{1}{2}$ y este extremo es $3$ para $a=\frac{2}{3}$. Como este extremo superior es estrictamente creciente en $a$, llegamos a que la solución al apartado (b) son los valores de $a$ en el intervalo $(\frac{1}{2},\frac{2}{3}]$. Obsérvese que el intervalo es abierto por la izquierda y cerrado por la derecha puesto que $I$ es abierto y no contiene a sus extremos.imagen
Si crees que el enunciado contiene un error o imprecisión o bien crees que la información sobre la procedencia del problema es incorrecta, puedes notificarlo usando los siguientes botones:
Informar de error en enunciado Informar de procedencia del problema
José Miguel Manzano © 2010-2024. Esta página ha sido creada mediante software libre